www.vorhilfe.de
- Förderverein -
Der Förderverein.

Gemeinnütziger Verein zur Finanzierung des Projekts Vorhilfe.de.
Hallo Gast!einloggen | registrieren ]
Startseite · Mitglieder · Impressum
Forenbaum
^ Forenbaum
Status VH e.V.
  Status Vereinsforum

Gezeigt werden alle Foren bis zur Tiefe 2

Navigation
 Startseite...
 Suchen
 Impressum
Das Projekt
Server und Internetanbindung werden durch Spenden finanziert.
Organisiert wird das Projekt von unserem Koordinatorenteam.
Hunderte Mitglieder helfen ehrenamtlich in unseren moderierten Foren.
Anbieter der Seite ist der gemeinnützige Verein "Vorhilfe.de e.V.".
Partnerseiten
Weitere Fächer:

Open Source FunktionenplotterFunkyPlot: Kostenloser und quelloffener Funktionenplotter für Linux und andere Betriebssysteme
Forum "Gruppe, Ring, Körper" - endliches ideal
endliches ideal < Gruppe, Ring, Körper < Algebra < Algebra+Zahlentheo. < Hochschule < Mathe < Vorhilfe
Ansicht: [ geschachtelt ] | ^ Forum "Gruppe, Ring, Körper"  | ^^ Alle Foren  | ^ Forenbaum  | Materialien

endliches ideal: Frage (beantwortet)
Status: (Frage) beantwortet Status 
Datum: 12:09 Mi 19.12.2012
Autor: Schadowmaster

moin,

Für eine Hausaufgabe brauche ich gerade folgende Aussage:

Aufgabe
Sei $R$ ein Ring (nicht zwingend kommutativ, aber mit $1$) und [mm] $\{0\} \neq [/mm] I$ ein einseitiges Ideal in $R$; oBdA rechtsseitig. Ist [mm] $|R|=\infty$, [/mm] so folgt [mm] $|I|=\infty$ [/mm]




Ich bin mir nicht wirklich sicher, ob die Aussage gilt.
Haben wir ein $a [mm] \in [/mm] I$, das kein Linksnullteiler ist, so ist $|aR|= [mm] \infty$ [/mm] und da [mm] $aR\subseteq [/mm] I$ gilt, wäre die Aussage damit gezeigt.
Aber was wäre, wenn $I$ nur Nullteiler enthält?
Mir will da leider weder ein Beweis für die Aussage noch ein Gegenbeispiel einfallen, weswegen ich mich über das eine oder das andere freuen würde.


lg

Schadow

        
Bezug
endliches ideal: Antwort
Status: (Antwort) fertig Status 
Datum: 08:23 Do 20.12.2012
Autor: hippias

Deine Vermutung gilt nicht: Sei etwa $R:= [mm] \IZ\times \IZ_{2}$ [/mm] mit komponentenweiser Addition und Multiplikation. $I:= [mm] \{0\}\times \IZ_{2}$ [/mm] is ein endliches Ideal, aber $R$ ist nicht endlich.

Bezug
Ansicht: [ geschachtelt ] | ^ Forum "Gruppe, Ring, Körper"  | ^^ Alle Foren  | ^ Forenbaum  | Materialien


^ Seitenanfang ^
ev.vorhilfe.de
[ Startseite | Mitglieder | Impressum ]